信号与系统习题与答案 下载本文

习题答案

1.1 选择题(每小题可能有一个或几个正确答案,将正确的题号填入[ ]内) 1.f(5-2t)是如下运算的结果————————( 3 ) (1)f(-2t)右移5 (2)f(-2t)左移5

55 (4)f(-2t)左移

221.2 是非题(下述结论若正确,则在括号内填入√,若错误则填入×)

(3)f(-2t)右移

1.偶函数加上直流后仍为偶函数。 ( √ ) 2. 不同的系统具有不同的数学模型。 ( × ) 3. 任何信号都可以分解为偶分量与奇分量之和。 ( √ ) 4.奇谐函数一定是奇函数。 ( × ) 5.线性系统一定满足微分特性 ( × ) 1.3 填空题 1.?(t)?cost??(t)

?(t?1)cos?0t?cos?0?(t?1)

?(t)?cos?0(t??)?cos(?0?)?(t)

(1?cost)?(t??)??(t?)

22?????(1?cost)?(t??2)dt? 1 ??????(t)?costdt? 1 ??

????t?(t)cos?0tdt? 1 ???(?)cos?0?d??u(t) ?(t?1)cos?0tdt?co?s0

??????t???(??1)co?s0?d??co?s0ut(? 1)2.?(t)?e?at??(t)

?(t)?e?t??(t)

?

t???e???(?)d??u(t)

[t2?e?2t]?(t?1)dt?1?e?2

????????(t)e?atdt? 1 1.4 简答题

1.信号f(t)如题图四所示,试求f?(t)表达式,并画出f?(t)的波形。

1 -1 -1 图四 -1f(t) 1 t 1f?(t) 1答案:因为 f(t)?t[u(t?1)?u(t?1)] 所以 f?(t)?u(t?1)?u(t?1)??(t?1)??(t?1) -1t2.f(t)波形如题图五所示,试写出其表达式(要求用阶跃信号表示)。

3 2 1 0 1 2 3 t f(t)

图五

答案:f(t)=3u(t)-u(t-1)-u(t-2)-u(t-3)

1.5 讨论以下系统是不是线性,时不变系统,并说明理由。

1.y(t)?2x(t)?3; (时不变、非线性) 2.y(n)?sin(t??2??n?)x(n); (线性、时变) 763.y(t)??x(??1)d?; (线性、时不变)

4.y(n)?m????x(m)。 (线性、时不变)

dy(t)4?2y(t)?2x(t),若x(t)?u(t), y(0?)?,解得完全响应dt3n2.1 选择题(每小题可能有一个或几个正确答案,将正确的题号填入( )内) 1.系统微分方程式

1y(t)=e?2t?1,(当t?0) 则零输入响应分量为——————————— ( 3 )

31?2t11?2t (1)e (2)e?

3334 (3)e?2t (4)?e?2t?1

32.已知f1(t)?u(t),f2(t)?e?atu(t),可以求得f1(t)*f2(t)?—————( 3 ) (1)1-e?at (2)e?at

11 (3)(1?e?at) (4)e?at

aa3.线性系统响应满足以下规律————————————( 1、4 )

(1)若起始状态为零,则零输入响应为零。 (2)若起始状态为零,则零状态响应为零。

(3)若系统的零状态响应为零,则强迫响应也为零。 (4)若激励信号为零,零输入响应就是自由响应。

4.若系统的起始状态为0,在x(t)的激励下,所得的响应为———( 4 ) (1)强迫响应;(2)稳态响应;(3)暂态响应;(4)零状态响应。

2.2 是非题(下述结论若正确,则在括号内填入√,若错误则填入×)

1.零输入响应就是由输入信号产生的响应。 ( × ) 2.零状态响应是自由响应的一部分。 ( × ) 3.若系统起始状态为零,则系统的零状态响应就是系统的强迫响应 ( × ) 4.当激励为冲激信号时,系统的全响应就是冲激响应。 ( × )

5.已知f1(t)?u(t?1)?u(t?1),f2(t)?u(t?1)?u(t?2),则f1(t)*f2(t)的非零值区间为(0,3)。 ( √ )

2.3 填空题 1.?(t)*e?t?e?t

?(t)?e?at?e?at

2.?(t?1)*cos?0t?cos?0(t?1)

?(t)*cos?0(t??)?cos?0(t??)

(1?cost)*?(t?)?1?cos(t?)

22d[u(t)*u(t)]?u(t) dtd[u(t)?tu(t)]?tu(t) dttd?u(t)*?u(?)d???tu(t)

?????dt?d?t[eu(t)*u(t)]?e?tu(t) dt4.已知f1(t)?u(t)?u(t?1),f2(t)?u(t?1)?u(t),则f1(t)*f2(t)的非零值区间为( -1 ,1 )

5.某线性时不变系统的阶跃响应g(t)?(1?e?2t)u(t), 为使其零状态响应

1yzs(t)?(1?e?2t?te?2t)u(t),其输入信号x(t)=(1?e?2t)u(t)

2??3.

dy(t)1?2y(t)?2x(t),若x(t)?u(t)解得完全响应y(t)?1?e?2tdt3?(当t≥0),则系统的起始状态y(0)= 4/3

7.一起始储能为零的系统,当输入为 u(t)时,系统响应为e?3tu(t),则当输入

6.已知系统方程式

为δ(t)时,系统的响应为?(t)?3e?3tu(t)

8.下列总系统的单位冲激响应 h(t)=h2(t)?h1(t)*h2(t)

x(t)

2.4 计算下列卷积

h1(t) ?h2(t) y(t)

1.s(t)?sint?u(t)*u(t?1) 答案:s(t)?[1?cos(t?1)]u(t?1) 2.s(t)?e?tu(t)?e?2tu(t) 答案:s(t)?(e?t?e?2t)u(t)

3.s(t)?E[u(t)?u(t?1)]*E[u(t)?u(t?3)],并画出s(t)的波形。 答案:s(t)?E2tu(t)?E2(t?1)u(t?1)?E2(t?3)u(t?3)?E2(t?4))u(t?4)

s(t) E2 0 1 2 3 4 t

4.f1(t)与f2(t)的波形如题图所示,计算卷积s(t)=f1(t)* f2(t),并画出s(t)的波形图。

2 1 0 1 t 0 2 t f1(t) f2(t)

S(t)20123t答案:s(t)?2tu(t)?2(t?1)u(t?1)?2(t?2)u(t?2)?2(t?3)u(t?3)

5.已知f1(t)如题图所示,f2(t)?e?tu(t),求卷积s(t)=f1(t)* f2(t),并画出s(t)波形。

2 1 1 答案:s(t)?u(?t?1)?[2?e?(t?1)]u(t?1)

f1(t) t

s(t)21-10123t116.已知f1(t)?u(t?1)?u(t?1),f2(t)??(t?1)??(t?1),f3??(t?)+?(t?)

22 (1)分别画出f1(t)、f2(t)及f3(t)的波形; (2)求s1(t)=f1(t)*f2(t),并画出s1(t)的波形; (3)求s2(t)=f1(t)*f3(t),并画出s2(t)的波形。

答案:(1)

f1(t)1-10-1(1)1tf2(t)(1)01t(1)f3(t)(1)t-1/201/2(2)s1(t)?u(t?2)?u(t?2)

(3)s2(t)?u(t?)?u(t?)?u(t?)?u(t?)

112.5 已知某系统的阶跃响应为g(t)?(?e?t?e?2t)u(t),试写出该系统的微分

22方程式。

答案:系统的冲击响应为:h(t)?(e?t?e?2t)u(t)

32121232d2y(t)dy(t)?3?2y(t)?x(t) 系统的微分方程式:2dtdt

2.6 某线性时不变系统在零状态条件下,当激励x1(t)= tu(t)时,响应y1(t)=eu(t), 试求当激励x2(t)=u(t)时,响应y2(t)的表达式。

答案:y2(t)??e?tu(t)??(t)

2.7 题图所示系统是由两个子系统级联而成的,两子系统的冲激响应分别为: h1(t)?t[u(t)?u(t?1)],h2(t)?u(t?1)?u(t?2) 试求总系统的冲激响应h(t),并画出h(t)的波形。

?t x(t) h1(t) h2(t) y(t)

(t?1)24t?t2?3[u(t?1)?u(t?2)]?[u(t?2)?u(t?3)] 答案:h(t)?h1(t)*h2(t)?22h(t)1/2023t12.8 已知某一阶线性时不变系统,当激励信号x(t)=u(t)时,全响应

?13?y(t)???e?2t?u(t),若已知系统的起始状态y(0?)?1,求系统的零输入响应

?22?yzi(t)与冲激响应h(t)。

答案:系统的零输入响应:yzp(t)?e?2tu(t)

冲激响应:h(t)??(t)?e?2tu(t)

2.9 一线性时不变系统的输入x(t)与零状态响应yzs(t)如题图所示: 1.求系统的冲激响应h(t);

2.当输入为图五所示的其它信号x1(t)及x2(t)时,画出系统的零状态响应的波形。

1 0 1 t x(t) 1 0 1 2 t 0 yzs(t) x1(t) 1 1 2 t 0 -1 x2(t) 1 2 t 答案:1. 系统的冲激响应:h(t)?u(t)?u(t?1)

3.1 选择题(每小题可能有一个或几个正确答案,将正确的题号填入( )内) 1.已知f(t)的频带宽度为Δω,则f(2t-4)的频带宽度为—————( 1 )

1 (1)2Δω (2)?? (3)2(Δω-4) (4)2(Δω-2)

22.已知信号f(t)的频带宽度为Δω,则f(3t-2)频带宽度为————( 1 )

(1)3Δω (2)Δω (3)(Δω-2) (4)(Δω-6) 3.理想不失真传输系统的传输函数H(jω)是 ————————( 2 )

?j?0t?j?t0?j?t0Ke (1) (2)Ke (3)Ke?u(???c)?u(???c)?

131313 (4)Ke?j?0t0 (t0,?0,?c,k为常数)

4.理想低通滤波器的传输函数H(j?)是——————————( 2 )

(1)Ke?j?t (2)Ke?j?t0[u(???C)?u(???C)]

0(3)Ke?j?0tK[u(???C)?u(???C)] (4)

j????t0,?0,?C,K,????均为常数?? ??5.已知:F1(j?)?F[f1(t)],F2(j?)?F[f2(t)]其中,F1(j?)的最高频率分量为

?1,F2(j?)的最高频率分量为?2,若对f1(t)?f2(t)进行理想取样,则奈奎斯特取样

频率fs应为(?2??1)————————————( 3 )

(1)2ω1 (2)ω1+ω2 (3)2(ω1+ω2) (4)(ω1+ω2) 6.已知信号f(t)?Sa(100t)?Sa2(60t),则奈奎斯特取样频率fs为——( 4 )

(1)

5012? (2)

120? (3)

100? (4)

60 ?7.若F1(j?)?F[f1(t)],则F2(j?)?F[f1(4?2t)]?—————————( 4 )

11? (1)F1(j?)e?j4? (2)F1(?j)e?j4?

2221? (3)F1(?j?)e?j? (4)F1(?j)e?j2?

2218.若对f(t)进行理想取样,其奈奎斯特取样频率为fs,则对f(t?2)进行取

3样,其奈奎斯特取样频率为————————( 2 )

(1)3fs (2)

11fs (3)3(fs-2) (4)(fs?2) 339.信号f(t)=Sa(100t),其最低取样频率fs为—————————( 1 )

?? (3) (4)

10020010.一非周期连续信号被理想冲激取样后,取样信号的频谱F(是——( 3 ) sjω)

(1)

100?? (2)

200(1)离散频谱; (2)连续频谱;

(3)连续周期频谱; (4)不确定,要依赖于信号而变化 11.图示信号f(t),其傅氏变换F[f(t)]?F(j?)?R(?)?jX(?),实部R(ω)的表示式为———————————————————( 3 )

? (1)3Sa(2ω) (2)3Sa()

2 (3)3Sa(ω) (4)2Sa(ω)

f(t) 2 1 -1 1 t

12.连续周期信号f(t)的频谱F(j?)的特点是———————( 4 )

(1)周期、连续频谱; (2)周期、离散频谱; (3)连续、非周期频谱; (4)离散、非周期频谱。

13.欲使信号通过线性系统不产生失真,则该系统应具有——————( 3、4 )

(1)幅频特性为线性,相频特性也为线性; (2)幅频特性为线性,相频特性为常数; (3)幅频特性为常数,相频特性为线性; (4)系统的冲激响应为h(t)?k?(t?t0)。

14.一个阶跃信号通过理想低通滤波器之后,响应波形的前沿建立时间tr与—————————————————( 4 )

(1)滤波器的相频特性斜率成正比; (2)滤波器的截止频率成正比; (3)滤波器的相频特性斜率成反比; (4)滤波器的截止频率成反比;

(5)滤波器的相频特性斜率和截止频率均有关系。

3.2 是非题(下述结论若正确,则在括号内填入√,若错误则填入×) 1.若周期信号f(t)是奇谐函数,则其傅氏级数中不会含有直流分量。 ( √ ) 2.奇函数加上直流后,傅氏级数中仍含有正弦分量。 ( √ ) 3.周期性冲激序列的傅里叶变换也是周期性冲激函数 ( √ ) 4.阶跃信号通过理想低通滤波器后,响应波形的前沿建立时间tr与滤波器的截 止频率成正比 ( × ) 5.周期性的连续时间信号,其频谱是离散的、非周期的 ( √ )6.非周期的取样时间信号,其频谱是离散的、周期的 ( × ) 3.3 填空题

1.已知F[f(t)]?F(j?),则

F [f(3t?3)]?F(13j??j?)e 3?1j??j52F [f(2t-5)]=F()e

22?1j??j32F [f(3-2t)] =F(?)e

22F[f(t)cos200t]=?F[j(??200)]?F[j(??200)]?

1?j(???0)??F[j(???0)]e?j(???0)? F [f(t??)cos?0t]?F[j(???0)]e212??F[f(t)ej?t]?F[j(???0)]

01?2?F [f(t)??(t?nT1)]??F[j(??n)]

T1n???T1n???? F-1[F(j?)e?j?t0]=f(t?t0)

j?tF ?1[F(j(???0)]?f(t)e0

2.已知F1(j?)?F[f1(t)],F2(j?)?F[f2(t)],其中:F1(j?)的最高频率分量为?1,

F2(j?)的最高频率分量为?2,且?2??1,则f(t)?f1(t)?f22(t)的最高频率分 量fm=2?2,若对f(t)进行取样,则奈奎斯特取样周期Ts=

? 2?23.若理想低通滤波器截止频率fc?1KHz,则阶跃信号通过该滤波器后响应的上 升时间tr= 1 毫秒 。

4.无失真传输系统,其幅频特性为H(j?)?K,相频特性为?(?)???t0; 理想低通滤波器的系统函数H(jω)=ke?j?t0[u(???0)?u(???0)]

5.已知F(j?)?F[f(t)],F(j?)的最高频率为fm,现对f(t)进行理想冲激取样,

1则取样信号fs(t)的傅氏变换Fs(j?)?F[fs(t)]?Tsn????F[j(??n?)],若要保

s?证能从fs(t)中恢复出原信号,则最大取样周期Tsmax=12fm。

6.无失真传输系统的系统函数H(jω)=ke?j?t0

7.已知f1(t)的频谱函数在(-500Hz,500Hz)区间内不为零,f2(t)的频谱

函数在(-1000Hz,1000Hz)区间内不为零,现对f1(t)与f2(t)相乘所得 的信号进行理想取样,则奈奎斯特取样频率为3000Hz。

8. 已知f(t)的最高频率分量fm为103Hz,则信号f(t)的最低取样率 fs=2?103Hz,则信号f(2t)的最低取样率fs=4?103Hz 9.已知g(t)????f(?)Sa[?c(t??)]d?和F[f(t)]=F(jω)

则G(jω)=F[g(t)]=

??F(j?)[u(???c)?u(???c)] ?c10.图示周期方波信号f(t)包含有哪些频率分量?

奇次谐波的正弦分量 粗略画出信号频谱图。

-T/2 |Cn|f(t) E/2 0 T/2 -E/2 t 0ω12ω13ω14ω15ω1ω

11.F?[?(t)*1]cos?0t???[?(???0)??(???0)]

已知F?sgn(t)??2j?1?,求 F?????j?sgn(?)

?t?12.已知信号f(t)的频谱函数在(-500Hz,500Hz)区间内不为零,现对f(t)

进行理想取样,则奈奎斯特取样频率为 1000 Hz。

13.周期信号f(t)如题图所示,若重复频率f=5KHz,脉宽??20?s,幅度E=10V,

则直流分量= 1 V。

E f(t) ? -T ? ??2 ?2 T t

j2t14.F[eu(t)]=??(??2)?1

j(??2) F [t]=2?j?'(?)。

3.4 已知某周期信号的傅里叶级数:

11f(t)?2E[cos?1t?cos3?1t?cos5?1t??]

35试画出f(t)的幅度频谱|Fn|~ω的图形。 答案:

|Fn|EE/3E/5?5?1?4?1?3?1?2?1??10?12?13?14?15?1??t的频谱Y(j?)?F[y(t)],3.5已知x(t)?E[u(t?1)?u(t?1)],求y(t)?x(t)cos200并画出y(t)的频谱图Y(jω)。 答案:

1Y(j?)?{X[j(??200?)]?X[j(??200?)]}?E[Sa(??200?)?Sa(??200?)]

2Y(j?)E199??200?201?200?0?3.6 求图示频谱函数F(jω)的傅里叶反变换,f(t)=F-1[F(jω)],并画出

f(t)的波形图。

F(jω) 1 -2 0 2 ω

答案:f(t)?2?Sa(2t)

f(t)2??2??20?t3.7 f1(t)与f2(t)的频谱如图所示,分别求f1(t)+f2(t),f1(t)*f2(t)及f1(t)·f2(t)的频谱表达式,并画频谱图。

F1(jω) 1 -5 0 5 ω -2 0 2 ω 2 F2(jω)

答案:F?f1(t)?f2(t)??F1(j?)?F2(j?), F?f1(t)?f2(t)??F1(j?)?F2(j?)

F?f1(t)?f2(t)??1F1(j?)?F2(j?) 2?F[f1(t)?f2(t)]31-5-2025?2F[f1(t)*f2(t)]-202?F4/?-3[f1(t)?f2(t)]-7037?

3.15 系统如题图(a)所示,低通滤波器的传输函数如题图(b)所示,已知

nx(t)?Sa(2?t),s(t)???(t?)

3n???? |H(jω)| 1 -2π 2π ω

x(t) 时域相乘 s(t) 滤波器 H(jω) (a) y(t) φ(ω) ω (b) 1 ω 2 1.求信号x(t)的频谱X(j?)?F[x(t)],并画出X(j?)~?图形; 2.求输出信号y(t),并粗略画出其波形。 答案: 1)X(j?)?1?u(??2?)?u(??2?)? 2X(j?)1/2?2?02??1??2)y(t)?3Sa?2?(t?)??3Sa(2?t??)

2??

y(t)31-1/2

4.1 是非题(下述结论若正确,则在括号内填入√,若错误则填入×)

1.若L[f(t)]?F(s),则L[f(t?t0)]?e?st0F(s) ( × )

01/23/2t?e?s?2.L ? ?sin(t?1) ( × )2??1?s?3.拉氏变换法既能求解系统的稳态响应,又能求解系统的暂态响应。( √ )

?1?4.2 求L[2???e?(?)d?]

?答案:L[2???e?(?)d?]=L[2u(t)]=

tt2 s

4.3 已知系统函数的极点为p1=0,p2=-1,零点为z1=1,如该系统的冲激响应的

终值为-10,求此系统的系统函数H(s)。 答案:H(s)?

10(s?1)

s(s?1)4.4 对于题图所示的RC电路,若起始储能为零,以x(t)作为激励,v2(t)作为响应,

+ x( t) - 0.5 F + (1) 2Ω v2(t) - 0 x(t) … 1 2 3 4 t

1.求系统的冲激响应h(t)与阶跃响应g(t),并画出h(t)及g(t)的波形; 2.若激励信号x1(t)?u(t)?u(t?1),求系统响应v2(t); 3.若激励信号x2(t)如题图所示,求系统响应v2(t)。 答案:1. h(t)??(t)?e?tu(t)

g(t)??h(?)d??e?tu(t)

??t

h(t) (1) g(t) 1 t

t

?1 2. v2(t)?g(t)?g(t?1)?e?tu(t)?e?(t?1)u(t?1)

?(t?n)u(t?n)] 3. v2(t)??h(t?n)??[?(t?n)?en?0n?0??

1F,t = 0以前开关位于“1”,电路2已进入稳定状态;t = 0开关从“1”倒向“2”,

4.5 系统如题图所示,L=1H,R=2Ω,C=

R E 1 L 2 i(t) R C

1.画出系统的s域模型; 2.求电流i(t)。 答案:1.

sL - LiL(0?) + R I (s) 1sCC

?其中:iL(0)??E R2. i(t)??

E?te(cost?sint)u(t) 24.6 有一一阶低通滤波器,当激励为sint u(t)时,自由响应为2e?3tu(t),求

强迫响应(设起始状态为零)。 答案: yp(t)?(?2cost?6sint)u(t)

4.7 电路如题图所示,x(t)为激励信号,以vc(t)作为响应。

x(t) + - 2Ω 1H + 1F vc(t) -

1.求该系统的系统函数H(s)及冲激响应h(t); 2.画出该系统的s域模型图(包含等效电源);

??3.求系统的起始状态iL(0),vc(0),使系统的零输入响应等于冲激响应;

4. 求系统的起始状态iL(0?),vc(0?),使系统对x(t)?u(t)的全响应仍为u(t)。

1s2?s?1s1 (s?1)2答案:1. H(s)??h(t)?te?tu(t)

2.

R + X(s)- -sL LiL(0) - + 1/sC ?vC(0- )+ s+ VC(s) - - ??3. (1) iL(0)?1A,vC(0)?0 ??4. iL(0)?0,vC(0)?1V

-

5.1 选择题(每小题可能有一个或几个正确答案,将正确的题号填入( )内)

amsm?am?1sm?1??a1s?a01.若一因果系统的系统函数为H(s)?,则有如下结

bnsn?bn?1sn?1??b1s?b0论—————————— ( 2 )

(1) 若bi?0(i?0,1,?n,且n?2),则系统稳定。

(2) 若H(s)的所有极点均在左半s平面,则系统稳定。

(3) 若H(s)的所有极点均在s平面的单位圆内,则系统稳定。 2.一线性时不变因果系统的系统函数为H(s),系统稳定的条件是—— (3、4 )

(1) H(s)的极点在s平面的单位圆内; (2) H(s)的极点的模值小于1;

(3) H(s)的极点全部在s平面的左半平面; (4) H(s)为有理多项式。

3.线性系统响应的分解特性满足以下规律————( 2、3 )

(1) 若系统的起始状态为零,则系统的自由响应为零; (2) 若系统的起始状态为零,则系统的零输入响应为零; (3) 若系统的零状态响应为零,则强迫响应亦为零; (4) 一般情况下,零状态响应与系统特性无关。

4.系统函数H(s)与激励信号X(s)之间——( 2 )

(1)是反比关系; (2)无关系; (3)线性关系; (4)不确定。 5.线性时不变系统输出中的自由响应的形式由——————( 1 )决定

(1)系统函数极点的位置; (2)激励信号的形式;

(3)系统起始状态; (4)以上均不对。

5.2 是非题(下述结论若正确,则在括号内填入√,若错误则填入×)

11.若已知系统函数H(s)?,激励信号为x(t)?e?2tu(t),则系统的自由响

s(s?1)应中必包含稳态响应分量。 ( √ )

2.强迫响应一定是稳态响应。 ( × ) 3.系统函数与激励信号无关 ( √ )

5.3 填空题

s1.已知系统函数H(s)?2,起始条件为:y(0?)?1,y?(0?)?0,则系统的

s?1零输入响应yzi(t)= ( cost?u(t) )

2.某线性时不变系统,当起始状态为y(0?)、激励信号为x(t)的情况下, 系

1?2ty(t)?eu(t),零状态响应为yzs(t)?(e?2t?1)u(t),若起统的零输入响应为zi2始状变为2y(0?)、激励信号变为

?2t?2(t?1)?1?(eu(t)???e?u(t?1))

1x(t?1),则系统的全响应为2123.已知系统函数H(s)=范围( ?1?k?1 ) 5.4 已知系统的微分方程为答案: H(s)?s, s?11,要使系统稳定,试确定k值的2s?(1?k)s?k?1dy(t)dx(t)?y(t)?,求系统函数H(s) dtdt5.5 已知某系统的系统函数H(s)=

1,

s2?2s?k?2若使系统稳定,求k值应满足的条件;

答案: k?2

5.6 电路如题图所示,t =0以前开关位于“1”,电路已进入稳态,t =0时刻开关转至“2”,以流经电阻上的电流作为响应。

2 + x(t) 1 10V 1F i(t) 1Ω

1.求系统函数H(s),画出零极点分布图,并说明系统是否稳定。

2.画出t≥0后的s域模型图(包含等效电源); 3.若激励x(t)=δ(t),求电流i(t)的零输入响应,零状态响应与全响

应,并指出全响应中的暂态响应与稳态响应分量。

答案:1. H(s)?

I(s)s? X(s)s?1j? -1 ? 0 ?

由于H(s)的极点-1在左半s平面,所以系统稳定。

2.

vC(0?)1 ssC- + + X(s)- - I(s) R

其中:vC(0?1)?10V

3. izs(t)??(t)?e?tu(t)

izi(t)??10e?tu(t)

i(t)?izi(t)?izs(t)??(t)?11e?tu(t)

i(t)即为暂态响应分量,无稳态响应分量。

5.7 给定系统的微分方程

dy(t)d(x)t )?2y(t)??2x(tdtdt 1.当激励x(t)为u(t)时,系统全响应y(t)为(5e-2t-1)u(t),求该

系统的起始状态y(0?)(要求用拉氏变换方法求);

2.求系统函数H(s),并画出系统的模拟结构框图;

3.画出H(s)的零极点图,并粗略画出系统的幅频与相频特性曲线。 答案:1. y(0?)?3

2.

Y(s)s?21?2s?1H(s)???X(s)s?21?2s?1

3.

j?

? ? 0 2 -2

5.8 系统如题图所示(设系统初始无储能),

X(s) + Σ - 1S1 S Y(s)

1.求系统函数H(s)?Y(s),并讨论系统的稳定性; X(s)2.粗略画出系统的幅频特性与相频特性曲线; 3.求系统的冲激响应与阶跃响应;

4.若激励信号x(t)?u(t)?u(t?1),求响应y(t),并指出暂态响应与稳定

响应各分量。

答案:1. H(s)?Y(s)1? X(s)s(s?1)由于H(s)的两极点p1?0,p2??1均在左半s平面,所以系统稳定。

2. j?

? ? 0 ? -1

?t3. h(t)?(1?e)u(t)

H(j?)?(?)??90???180?g(t)?(t?1?e?t)u(t)

4. y(t)?g(t)?g(t?1)?(t?1?e?t)u(t)?(t?2?e?(t?1))u(t?1)

其中(t?1)u(t)?(t?2)u(t?1)为稳态响应分量,e?tu(t)?e?(t?1)u(t?1)为暂态响应分量。

t28?135.9 如图(a)所示系统,当x(t)??(t)时,全响应y(t)??(t)?eu(t),

39?并已知电容上的起始电压v(0)?1V

1.求系统的零输入响应yzi(t)及h(t)和g(t),并画出波形;

1Ω + x(t) (a) 2Ω 1F + y(t) (1) ? 0 T 2T 3T 4T (b) t x2(t)

2.粗略画出系统的幅频特性及相频特性曲线;

3.若激励信号x1(t)?u(t)?u(t?1)时,求系统的零状态响应yzs(t); 4.若激励信号x2(t)如图(b)所示,求系统的零状态响应yzs(t)。

t2?13答案:1. yzi(t)??eu(t)

3t22?13?(t)?eu(t) h(t)=39t2?13g(t)?eu(t)

32.

H(j?)2/3 ?(?)90??1t?(t?1)2?1233u(t?1) 3. yzs(t)?g(t)?g(t?1)?eu(t)?e33(t?nT)?2?2?13y(t)?h(t?nT)??(t?nT)?eu(t?nT)?? ??4. zs9n?0n?0?3????

5.10 某系统如题图所示,已知Y(s)=X(s),

X(s) Σ H1(s) s s?2Y(s) Ks

1.求H1(s),并画出H1(s)的结构框图;

2.若使H1(s)是稳定系统的系统函数,求K值范围;

3.当K=1时,写出系统H1(s)的频响特性H1(jω)的表示式,并粗略画出幅频特性与相频特性曲线。 答案:1. Hs?21(s)?s?K 2. K?0

3. 当K=1时,Hj??21(j?)?j??1 H(j?) 2 ?(?) ?

1

?

6.1 选择题(每小题可能有一个或几个正确答案,将正确的题号填入( )1.?(n)与u(n)之间满足如下关系——————— ( 2、3、4 ) ?? (1)u(n)??k) (2)u(n)?n?k)

k??(n?????(k?0 (3)?(n)?u(n)?u(n?1) (4)?(n)?u(?n)?u(?n?1)

6.2 填空题

1.?(t)与u(t)及?(n)与u(n)之间满足以下关系:

?(t)= ( du(t)tdt ), u(t)= ( ????(?)d? )

? ?(n)? ( u(n)?u(n?1) ), u(n)? ( ??(n?k) )

k?02.u(n)*[?(n)??(n?1)]? ( ?(n) ) u(n)*u(n?1)? ( nu(n) )

?(n)*u(n)? (

u(n) )

u(n)*u(n)? ( (n?1)u(n) )

6.3 计算下列卷积

1.s(n)?[u(n)?u(n?5)]?0.5nu(n)并画出s(n)的图形

内) 答案: s(n)?2(1?0.5n?1)?u(n)?u(n?5)??2(0.5n?4?0.5n?1)u(n?5)

?2(1?0.5n?1)u(n)?2(1?0.5n?4)u(n?5)

s(n)1012?3456n2.s(n)?0.6nu(n)*0.6nu(n)

答案: s(n)?(n?1)0.6nu(n)

3.已知两序列x1(n)、x2(n)如题图所示,试求y(n)= x1(n)* x2(n),并画出y(n)的图形。

1 -1 0 1 2 3 n x1(n) -1 -1 1 2 x2(n) 2 1 3 n

答案: y(n)???(n)??(n?1)??(n?2)?????(n?1)?2?(n?1)??(n?2)????(n?1)??(n)??(n?1)?3?(n?2)?3?(n?3)??(n?4)

y(n) 3 1 -1 -1 -1 1 2 3 4 3 1 n

6.4 已知一离散系统的差分方程为:y(n)-2y(n-1)=x(n),已知,y(-1)

=3,求零输入响应yzi(n)。 答案:yzi(n)?6?2n(n?0)

6.5 已知一因果离散系统的差分方程为:(yn)- (yn-1)-2y(n-2)=(xn)+2x(n-2),已知y(-1)=2,y(0)=2,x(n)=u(n),求零输入响应yzi(n)与零状态响应 yzs(n)。 答案:yzi(n)?2n?1?(?1)n(n?0)

13?13???yzs(n)??(n)?2?(n?1)??2n?1?(?1)n??u(n?2)??2n?1?(?1)n??u(n)

22?22???

7.1 选择题(每小题可能有一个或几个正确答案,将正确的题号填入( )内)

11.已知Z变换Z[x(n)]?,收敛域z?3,求逆变换得x(n)为——( 1 )

1?3z?1 (1)3nu(n) (2)3nu(n?1) (3)?3nu(?n) (4)?3?nu(?n?1) 2.已知Z变换Z[x(n)]?1,收敛域z?3,求逆变换得x(n)为—( 4 ) ?11?3z (1)3nu(n) (2)3?nu(?n) (2)?3nu(?n) (4)?3nu(?n?1)

3.一个因果稳定的离散系统,其H(z)的全部极点须分布在z平面的———

—( 2、4 )

(1)单位圆外 (2)单位圆内 (3)单位圆上

(4)单位圆内(含z=0) (5)单位圆内(不含z=0)

7.2 是非题(下述结论若正确,则在括号内填入√,若错误则填入×)

1z1.已知X(z)?,收敛域为?z?2,其逆变换

12(z?)(z?2)2n?2?n?1? Z[X(z)]???2u(?n?1)???u(n)? ( √ )

3??2?????12.离散因果系统,若H(z)的所有极点在单位圆外,则系统稳定 (× )

3.离散因果系统,若系统函数H(z)的全部极点在z平面的左半平面,则系统稳定 ( × ) 4.离散系统的零状态响应是激励信号x(n)与单位样值响应h(n)的卷积。 (√ )

7.3 填空题 1.求Z变换

12z???1?n?2 ),收敛域为 ( z?1 ) Z???u(n)??(n)?= (

12????2??z?2Z??(n?1)??(n?1)?= ( z?z?1 ),收敛域为 ( 0?z?? )

2.求逆Z变换

1] = ( u(n?1) ), (|z|>1) Z?1[z?1Z?1[?11n?111] = ( ()u(n?1) ) (|z|?)

22z?12?10z2?n5[1?(?1)]u(n) ) (|z|>1) Z? = ( ??(z?1)(z?1)?1?10znn??8(?0.5)?20u(n)?4(2)u(?n?1) ) (1<|z|<2) = ( ???3?(z?0.5)(z?1)(z?2)?13.已知变换Z[x(n)]? ?11?3zn若收敛域|z|>3 求逆变换得x(n)= ( 3u(n) )

Z-1??n 若收敛域|z|<3, 求逆变换得x(n)= ( ?3u(?n?1) )

4.已知X(z)=

z z?1若收敛域|z|>1 求逆变换得x(n)= ( u(n) ) 若收敛域|z|<1, 求逆变换得x(n)= ( ?u(?n?1) )

z5.已知变换Z[x(n)]?

(z?1)(z?2)若收敛域|z|>2, 求逆变换得x(n)= ( ?2n?1?u(n) ) 若收敛域|z|<1, 求逆变换得x(n)= ( ?1?2n?u(?n?1) ) 若收敛域1<|z|<2, 求逆变换得x(n)= ( ?u(n)?2nu(?n?1) )

?1.5z6.已知X(z)?2

z?2.5z?1n?un( ) ) 0.?5?n2?若收敛域|z|>2, 求逆变换得x(n)= ( ???n 若收敛域0.5<|z|<2, 求逆变换得x(n)= ( ?0.5?u(n)?2u(?n?1) )

n7.已知x(n)?e?anu(n)?2nu(?n?1)(a?0),

则X(z)= (

zz? ) , 收敛域为( e?a?z?2 ) ?az?2z?e8.已知x(n)?(0.5)nu(n)?eanu(?n?1)(a?0),

则X(z)= (

zz?, 收敛域为( 0.5?z?ea ) a )z?0.5z?e9.设x1(n)是一个长度为N的因果序列,其Z变换为X1(z),

1?z?N(N?1)X1(z) )则?x1(n?kN)的Z变换X(z)= ( , 收敛域为( z?0 ) ?N1?zk?0N10. 设x1(n)是一个长度为N的因果序列,其Z变换为X1(z),

?1x(n?kN)X1(z) )则?1的Z变换X(z)= ( , 收敛域为( z?1 ) ?N1?zk?011.设某因果离散系统的系统函数为H(z)? ( a?1 )

z,要使系统稳定,则a应满足 z?a12.已知系统的单位样值信号h(n),试判断系统的因果性与稳定性 0.5nu(n) ( 因果、稳定 )

2nu(-n-1) ( 非因果、稳定 ) 2n[u(n)-u(n-5)] ( 因果、稳定 )

?1?z???n?3??1?13.已知x(n)???[u(n)?u(n?8)],则X(z)= ( 1? ) 7?3??z?z??3??88收敛域为 ( z?0 ),并在z平面上画出其零极点图。

jIm(z) ? (7) 1 3Re(z)

7.4 已知:x(n)=a|n|,(-∞ < n < ∞),讨论a在什么条件下,X(z)=Z[x(n)]存在,并在存在的条件下,求出X(z),并标出其收敛域。 答案: 当a?1时,

zz??z?az?1(z?a)(z?1)aa1(a?)za1 aX(z)?a?z?

7.5 某因果离散时间系统由两个子系统级联而成,如题图所示,若描述两个子系统的差分方程分别为:

y1(n)?0.4x(n)?0.6x(n?1)

1y(n)?y(n?1)?y1(n)3H1(z) y1(n)

x(n) H2(z) y(n)

1.求每个子系统的系统函数H1(z)和H2(z); 2.求整个系统的单位样值响应h(n);

3.粗略画出子系统H2(z)的幅频特性曲线; 4.画出整个系统的结构框图。

23(z?)2答案:1. H1(z)?0.4?0.6z?1?5zH2(z)?1z?111?z?1z?33nz?0

z?1 3n2?1?3?1?2. h(n)???u(n)???5?3?5?3?n?1211?1?u(n?1)??(n)???u(n?1)

155?3?3.

jIm(z) 0 1 3? Re(z)

7.6 已知一因果离散系统的结构框图如题图所示,

求系统函数H(z)及系统的差分方程。

x(n) Σ -1 -0.16 z-1 z-1 2 Σ y(n)

1?2z?1答案: H(z)?

1?z?1?0.16z?2y(n)?y(n?1)?0.16y(n?2)?x(n)?2x(n?1)

7.7 某因果离散系统的结构框图如题图所示,

x(n) Σ Σ z-1 y(n) ?k 3?k 4

1.写出该系统的系统函数H(z); 2.k为何值时,该系统是稳定的?

13.如果k=1,x(n)=δ(n)-()nu(n),试求y(n);

4k?1kzz?答案: 1. H(z)?4?4

kk1?z?1z?331? 2.当k?3时,系统稳定。

1?1? 3. 当k?1时, y(n)?????4?3?n?1u(n?1)

1117.8 已知一因果离散系统,当输入x(n)?()nu(n)?()n?1u(n?1)时,零状态

2421响应为:yzs(n)?()nu(n),

3 1.求该系统的系统函数H(z)及单位样值响应h(n);

2.求该系统的差分方程;

3.画出该系统的直接型结构框图。

1z(z?)Yzs(z)2H(z)??答案: 1.

X(z)(z?1)(z?1)43n??1?n?1??h(n)??3???2???u(n)

?3?????4??z?1 32. y(n)?3.

711y(n?1)?y(n?2)?x(n)?x(n?1) 12122 x(n) 112Σ ?71 12z-1 ? 2Σ y(n) z-1

7.9 已知因果离散系统的差分方程为:y(n)?1y(n?1)?x(n) 2 1.画出系统的结构框图;

2.求系统的单位样值响应h(n),并画出h(n)的图形;

??1?n?1?n?3.若系统的零状态响应为yzs(n)?2???????u(n),求激励信号x(n),

23????????并指出yzs(n)中的自由响应,强迫响应,稳态响应及暂态响应各分量; 4.画出系统函数H(z)的零极点分布图。

答案:1.

x(n) Σ 1/2 y(n) z-1 ?1?2. h(n)???u(n)

?2?nh(n)1214183411?1?x(n)? 3. ??3?3?nn?1?1?u(n?1)???u(n?1)

?3?nn0125?n?1??1?yzs(n)中2??u(n)为自由响应,?2??u(n)为强迫响应。

?2??3?. 4.

jIm(z)

? Re(z) 0 0.5

7.10 已知离散因果系统的差分方程为

16y(n?2)?x(n)?x(n?1) y(n)?y(n?1)?5251.求出系统函数H(z),注明收敛域,讨论系统的稳定性; 2.试画出该系统的直接型结构图; 3.若已知x(n)=u(n),求系统的零状态响应yzs(n);

1?z?1z2?z?答案:1. H(z)?1?16?2231?z?z(z?)(z?)52555z?3 5由于两极点和?均在单位圆内,系统又为因果系统,所以该系统是稳定的。 2.

x(n) 6252535Σ ? 1 5z-1 Σ ?1 y(n) z-1

??2?n?1?3?n?1? 3. yzs(n)?????????u(n)

?5????5???

7.11 系统如题图所示

x(n) z-1 z-1 Σ

1.求系统函数H(z),并画出H(z)的极零点分布图;

2. 若激励x(n)??(n?1)??(n)??(n?1),求系统的零状态响应yzs(n),并画出yzs(n)波形。 答案:1. H(z)?1?z?z?1?2y(n) z2?z?1 ?z22. yzs(n)??(n?1)?2?(n)?3?(n?1)?2?(n?2)??(n?3)

yzs(n)12321

n0?1123

7.12 一线性时不变离散系统的系统函数H(z)的零极点分布图如题图所示,

jIm(z) × × -0.5 0 0.2 × 2 Re(z) 3

1.写出该系统的系统函数H(z)的表达式;

2.指出该系统函数可能有的四种收敛域,并将四种收敛域分别表示在z平

面上;

3.讨论上述四种收敛域所对应的各系统的稳定性与因果性。

4. 讨论上述四种收敛域情况下,哪些极点对应的响应为右边序列,哪些极点

对应的响应为左边序列? 答案:1. H(z)?Az?3

(z?0.2)(z?0.5)(z?2)2. z?0.2,0.2?z?0.5,0.5?z?2,z?2,分别如下图所示: jIm(z)jIm(z)?0.5??0.2?12 3Re(z)?0.5??0.2?12 3Re(z) jIm(z)jIm(z) 0.2? ??0.5 ? 23Re(z) 0.2??0.5? 2 ? 3Re(z)3. 对于z?0.2:非因果、非稳定;对于0.2?z?0.5:非因果、非稳定; 对于0.5?z?2:非因果、稳定;对于z?2:因果、非稳定; 4. 对于z?0.2:3个极点所对应的响应均为左边序列;

对于0.2?z?0.5:极点0.2所对应的响应为右边序列,极点-0.5和2所对应的响应为左边序列;

对于0.5?z?2:极点0.2和-0.5所对应的响应为右边序列,极点2所对应的响应为左边序列;

对于z?2:3个极点所对应的响应均为右边序列;

7.13知一因果离散系统的差分方程为:

y(n)- y(n-1)-2y(n-2)=x(n)+2x(n-2)

已知y(-1)=2,y(0)=2,x(n)=u(n),利用Z变换法求零输入响应yzi(n)与零状态响应yzs(n)。

3??n?11ny(n)?2?(?1)?u(n) 答案: zs??22??yzi(n)?2n?1?(?1)n

(n?0)